Basis for 2x2 Matrix: Understand the Concepts

  • Thread starter Thread starter math2010
  • Start date Start date
  • Tags Tags
    Basis Matrix
Click For Summary
A basis for the space of 2x2 matrices consists of four specific matrices, which are the standard basis matrices for M2,2. The identity matrix is not a basis by itself, as it does not span the entire space of 2x2 matrices, which has a dimension of four. The correct basis includes matrices that represent each position in a 2x2 matrix. Understanding that the space is spanned by four linearly independent matrices is crucial for grasping the concept of basis in this context. The discussion emphasizes the importance of recognizing the dimensionality of the space when determining a basis.
math2010
Messages
19
Reaction score
0

Homework Statement



What is a basis for the space of 2 x 2 matrices.


The Attempt at a Solution



I don't understand how to this at all. Is the 2x2 identity matrix abasis for 2x2 matrices? Because it's linearly independent and spanning the space? Can anyone explain?
 
Physics news on Phys.org
math2010 said:
Is the 2x2 identity matrix abasis for 2x2 matrices? Because it's linearly independent and spanning the space? Can anyone explain?

Yeap. Any 2x2 matrix in R^2 can be spanned by {(1,0),(0,1)} which is I2
 
math2010 said:

Homework Statement



What is a basis for the space of 2 x 2 matrices.


The Attempt at a Solution



I don't understand how to this at all. Is the 2x2 identity matrix abasis for 2x2 matrices? Because it's linearly independent and spanning the space? Can anyone explain?
I disagree with what Cronxeh said. The space of 2x2 matrices has dimension 4, so cannot be spanned by two vectors, let alone two vectors in R2.

The standard basis for M2,2 consists of these four matrices:
[1 0]
[0 0]

[0 1]
[0 0]

[0 0]
[1 0]

[0 0]
[0 1]
 
Question: A clock's minute hand has length 4 and its hour hand has length 3. What is the distance between the tips at the moment when it is increasing most rapidly?(Putnam Exam Question) Answer: Making assumption that both the hands moves at constant angular velocities, the answer is ## \sqrt{7} .## But don't you think this assumption is somewhat doubtful and wrong?

Similar threads

  • · Replies 18 ·
Replies
18
Views
2K
  • · Replies 7 ·
Replies
7
Views
3K
Replies
15
Views
2K
  • · Replies 4 ·
Replies
4
Views
3K
  • · Replies 1 ·
Replies
1
Views
2K
Replies
4
Views
2K
  • · Replies 15 ·
Replies
15
Views
3K
Replies
5
Views
3K
Replies
2
Views
2K
  • · Replies 5 ·
Replies
5
Views
7K